Jun 9

本文转载自xida博客朝花夕拾,原文网址:http://xidalapuda.diandian.com/post/2014-03-02/40061132729

 

Jordan 标准形定理是线性代数中的基本定理,专门为它写一篇长文好像有点多余:这方面的教材讲义实在是太多了!一个陈旧的定理还能写出什么新意来呢?

理由有两个。第一个原因是我曾经在给学生讲这个定理的时候,突然发现不知道该怎么启发学生为好。虽然我知道 Jordan 标准形定理的很多种证法,照念几个不在话下,但是感觉有点疙疙瘩瘩的:怎么才能说清定理背后的想法,让学生觉得定理的成立是顺理成章的呢?于是我知道我对这个定理的理解还有模糊的地方。

第二个原因是 Jordan 块有一个重要的代数性质是通常教材中不讲的,而这个性质是代数学中一类重要而常见的性质的雏形,这就是不可分解性。与之对应的是可对角化的线性变换的完全可约性。从一开始就让学生接触这些现象是有好处的。



~~~~~~~~~~~~~~~~~~~~~~~~~~~~~~~~~~~~~~~~~~~~~~~~~~~~~~~~~



我们从中学就知道整数环和多项式环有唯一因子分解定理:每个整数可以唯一地分解为素数的乘积,每个(域上的)多项式可以唯一地分解为不可约多项式的乘积。在数学里面有很多这样的唯一分解定理,而我们现在想知道:有没有所谓的 "线性变换的唯一分解定理" 呢?可以猜测如果有这样的定理存在,那么大概可以表述为如下的样子:



线性变换的唯一分解定理(粗糙的版本):设 $V$ 是域 $F$ 上的有限维向量空间,$A$ 是 $V$ 上的线性变换,则 $A$ 可以唯一地分解为若干个 "简单的" 线性变换的组合,而且这些 “简单的” 线性变换本身不能再分解。



这个表述很不清楚,整数和多项式的分解就是表示为因子的乘积,那么什么是线性变换的分解呢?什么又是不可分解的线性变换呢?正确的概念是直和:



设 $T$ 是 向量空间 $V$ 上的线性变换,如果 $V$ 可以分解为一些非平凡的子空间的直和 $V=V_1\oplus\cdots\oplus V_k$,使得每一个 $V_i$ 都是 $T-$ 不变的子空间,则称 $T$ 是可以分解的; 如果 $V$ 不存在这样的分解,则称 $T$ 是不可分解的线性变换。



这样我们就可以比较准确的表述线性变换的唯一分解定理了:



线性变换的唯一分解定理(修正的版本):设 $V$ 是域 $F$ 上的有限维向量空间,$T$ 是 $V$ 上的线性变换,则 $T$ 可以唯一地分解为若干个不可分解的线性变换的直和。



这里有一个很严重的问题需要说明:在一般的域 $F$ 上 研究 "不可分解" 的线性变换是一个棘手的多的问题,这个问题的解决要用到我们后面要学的有理标准形,而在复数域上问题就简单很多,这就是 Jordan 标准形做的事情。所以在本文中,域 $F$ 都假定为复数域 $\mathbb{C}$。



那么什么样的线性变换算是不可分解的线性变换呢?



最简单也是最重要的例子就是移位算子:假设 $T$ 在 $V$ 的一组基 $\{v_1,\cdots,v_n\}$ 的作用是一个向右的移位:

\[ T:\quad v_n\rightarrow v_{n-1}\rightarrow\cdots\rightarrow v_1\rightarrow0.\]

则称 $T$ 是一个移位算子。$T$ 在这组基下的矩阵

\[J_0=\begin{pmatrix}0&1&&\\&\ddots&\ddots&\\&&0&1\\&&&0\end{pmatrix}.\]

$J_0$ 叫做特征值为 0 的 Jordan 块。注意 $T$ 是一个幂零算子:$T^n=0$,它仅有唯一的特征值 0。



当然需要说明移位算子 $T$ 确实是不可分解的线性变换。如果 $V=W\oplus N$ 为两个非平凡 $T-$ 不变子空间的直和,则 $T$ 在 $W$ 和 $N$ 上各有一个特征值为 0 的特征向量,因此齐次线性方程组 $TX=0$ 的解空间至少包含两个线性无关的向量。但是 $T$ 的秩是 $n-1$,因此 $TX=0$ 的解空间是 1 维的,这就导致了矛盾。



用同样的方法可以说明给移位算子 $T$ 加上一个数乘变换以后得到的仍然是不可分解的线性变换:设 $\lambda\in\mathbb{C}$,$S=T+\lambda I$,则 $S$ 也是不可分解线性变换,其对应的矩阵

\[J_\lambda=\begin{pmatrix}\lambda&1&&\\&\ddots&\ddots&\\&&\lambda&1\\&&&\lambda\end{pmatrix}\]

叫做特征值为 $\lambda$ 的 Jordan 块。



现在我们已经找到了一族不可分解的线性变换,那么它们是否就构成了全部的线性变换呢?答案是肯定的,这就是 Jordan 标准形定理:



Jordan 标准形定理:设 $T$ 是 $\mathbb{C}$ 上有限维向量空间 $V$ 上的线性变换,则存在 $V$ 的一组基使得 $T$ 在这组基下的矩阵为 Jordan 块的直和:\[T=J_{\lambda_1}\oplus\cdots\oplus J_{\lambda_r}.\]这种分解是唯一的,意思是如果存在 $V$ 的另一组基使得 $T$ 的矩阵也是 Jordan 块的直和 \[T=J_{\mu_1}\oplus\cdots\oplus J_{\mu_s},\]则 $r=s$ 且适当重排后有 $J_{\lambda_i}=J_{\mu_i}$。



定理的结论包含存在性和唯一性两部分,我们先来处理存在性的证明。



~~~~~~~~~~~~~~~~~~~~~~~~~~~~~~~~~~~~~~~~~~~~~~~~~~~~~~~~~~



第一步:转化为幂零的情形



定理 【广义特征子空间分解】:设 $T$ 的特征多项式为 $f(x)$,而且 $f(x)$ 在复数域上分解为一次因式的乘积\[f(x)=(x-\lambda_1)^{n_1}\cdots(x-\lambda_k)^{n_k},\]

这里的 $\lambda_i$ 互不相同。令 $V_i=\ker (T-\lambda_i I)^{n_i}$,则每个 $V_i$ 都是 $T-$ 不变子空间而且\[ V=V_1\oplus\cdots\oplus V_m.\]

这样就把 $V$ 分解为一些不变子空间 $V_i$ 的直和,$T$ 限制在每个 $V_i$ 上只有单一的特征值 $\lambda_i$。



证明:显然 $V_i$ 都是 $T-$ 不变子空间。要证明 $V$ 是它们的直和,我们先从一个特别的结论开始:



对每个 $1\leq i\leq k$ 都存在多项式 $\pi_i(x)$ 使得 $\pi_i(x)\equiv1\mod (x-\lambda_i)^{n_i}$,但是对其它 $j\ne i$ 有 $\pi_i(x)\equiv0\mod (x-\lambda_j)^{n_j}$。线性变换 $\pi_i(T)$ 不是别的,正是 $V$ 到子空间 $V_i$ 的投影。



由于所有 $(x-\lambda_i)^{n_i}$ 的根互不相同,因而两两互素,所以根据中国剩余定理满足要求的 $\pi_i(x)$ 是存在的。显然 $\pi_i(T)$ 在 $V_i$ 上是恒等变换,而在其余的 $V_j\ne V_i$ 上是 0。$\pi(x)=\pi_1(x)+\cdots+\pi_k(x)$ 模任何 $(x-\lambda_i)^{n_i}$ 都是 1,因此 $\pi(x)-1$ 可以被 $T$ 的特征多项式 $f(x)$ 整除,从而 $\pi(T)-I$ 在 $V$ 上是零变换,这就证明了 $\pi(T)$ 是 $V$ 上的恒等变换。对任何 $v\in V$,\[v=\pi(T)v=\pi_1(T)v+\cdots+\pi_k(T)v.\]

我们来说明 $\pi_i(T)v\in V_i$,从而 $V=V_1+\cdots+V_k$。这是因为 $(x-\lambda_i)^{n_i}\pi_i(x)$ 可以被 $f(x)$ 整除,因此 $(T-\lambda_i)^{n_i}\pi_i(T)v=0$,这就证明了 $\pi_i(T)v\in V_i$。



我们再来说明这是直和。如果 $v_i\in V_i$ 满足 $v_1+\cdots+v_k=0$,用 $\pi_i(T)$ 作用在左边得到(根据前面的分析,$\pi_i(T)$ 在 $V_i$ 上是恒等变换而在其它 $V_j$ 上是 0)

\[\pi_i(T)v_1+\cdots+\pi_i(T)v_k=\pi_i(T)v_i=v_i=0,\]

由 $i$ 的任意性得到 $v_1=\cdots=v_k=0$,这就证明了是直和。



用中国剩余定理来构造特殊的算子(通常是给定的算子 $T$ 的多项式)是一个普遍而重要的技巧,这里的证明也许有点高端但却是最简洁的。

现在我们只需要考虑单个子空间 $V_i$。令 $N=T-\lambda_i$,则 $N$ 在 $V_i$ 上是幂零线性变换:$N^{n_i}=0$,这样问题归结为分析幂零线性变换 $N$ 的结构。

幂零线性变换更简单的原因是它可以表示为移位算子的直和,而移位算子的结构非常简单。

~~~~~~~~~~~~~~~~~~~~~~~~~~~~~~~~~~~~~~~~~~~~~~~~~~~~~~~~~~



第二步:对幂零变换的情形加以证明



设 $N$ 是 $V$ 上的幂零线性变换,要证明存在 $V$ 的一组基,使得 $N$ 的矩阵是若干 Jordan 块的和。注意一个 Jordan 块对应的是一个移位轨道\[ v\rightarrow Nv\rightarrow \cdots \rightarrow N^kv\rightarrow 0.\]

我们要证明存在若干条这样的互不相交的轨道,这些轨道所包含的全部非零向量构成 $V$ 的一组基。



这一步的证明方法很多,但是相差不是很大,具体喜欢那种要看个人主观,这里介绍的是最简单也是最容易被初学者接受的一种。



对 $V$ 的维数 $\dim V$ 归纳,$\dim V=1$ 时显然结论成立。

现假设结论对所有维数小于 $\dim V$ 的向量空间成立,我们考虑 $V$ 的像空间 $N(V)$。这是一个 $N-$ 不变子空间,且由于 $N$ 是幂零线性变换所以 $\dim N(V)<\dim V$,所以可以对子空间 $N(V)$ 使用归纳假设:存在 $N(V)$ 的一组基如下,它们构成 $q$ 条不相交的轨道 $\mathcal{O}_1,\cdots,\mathcal{O}_q$:

\[\begin{array}{l}&v_{1,1}\rightarrow v_{1,2}\rightarrow\cdots\rightarrow v_{1,n_1}\rightarrow 0.\\&v_{2,1}\rightarrow v_{2,2}\rightarrow\cdots\rightarrow v_{2,n_2}\rightarrow 0.\\&\cdots\cdots\cdots\\& v_{q,1}\rightarrow v_{q,2}\rightarrow\cdots\rightarrow v_{q,n_q}\rightarrow 0.\end{array}\]

由于 $v_{i,1}\in N(V)$ 因此可以设 $v_{i,1}=Nw_i$,从而我们得到一组更长的轨道(就是在前面加上一项)

\[\begin{array}{l}&w_1\rightarrow v_{1,1}\rightarrow v_{1,2}\rightarrow\cdots\rightarrow v_{1,n_1}\rightarrow 0.\\&w_2\rightarrow v_{2,1}\rightarrow v_{2,2}\rightarrow\cdots\rightarrow v_{2,n_2}\rightarrow 0.\\&\cdots\cdots\cdots\\&w_q\rightarrow v_{q,1}\rightarrow v_{q,2}\rightarrow\cdots\rightarrow v_{q,n_q}\rightarrow 0.\end{array}\]

那么这些新轨道包含的向量是否构成 $V$ 的一组基?答案是我们还要补上一些在 $V$ 中长度是 1,但是在 $N(V)$ 中 "消失" 了的轨道:注意 $\{v_{1,n_1},\cdots,v_{q,n_q}\}$ 是 $\ker N$ 中的线性无关元,但是 $\ker N$ 还可能有其它的基向量。将它们扩充为 $\ker N$ 的一组基

\[ \{ v_{1,n_1},\cdots,v_{q,n_1}\}\cup \{ w_{q+1},\cdots,w_{K}\}\quad K=\dim\ker N.\]

从而我们最终得到下面的轨道图:

\[\begin{array}{r}\mathbf{w_1\rightarrow} v_{1,1}\rightarrow v_{1,2}\rightarrow\cdots\rightarrow v_{1,n_1}\rightarrow 0.&\\\mathbf{w_2\rightarrow} v_{2,1}\rightarrow v_{2,2}\rightarrow\cdots\rightarrow v_{2,n_2}\rightarrow 0.&\\\cdots\cdots\cdots&\\\mathbf{w_q\rightarrow} v_{q,1}\rightarrow v_{q,2}\rightarrow\cdots\rightarrow v_{q,n_q}\rightarrow 0.&\\\mathbf{w_{q+1}\rightarrow} 0.&\\\cdots\cdots&\\\mathbf{w_K\rightarrow0}.\end{array}\]

你可以看到 $w_{q+1},\ldots,w_K$ 正是那些在 $V$ 中长度为 1,但是在 $N(V)$ 中消失了的轨道。

最后只剩下验证这些向量确实构成 $V$ 的一组基。显然这些向量一共有 $\dim N(V)+\dim\ker N=\dim V$ 个,所以只要说明它们是线性无关的。

假设有线性关系

\[\cdots+(c_0w_i+c_1v_{i,1}+\cdots+c_{n_i}v_{i,n_i})+\cdots+\sum_{j=q+1}^K d_jw_j=0,\]

我们要说明出现在上式中的所有系数都是 0。左边用 $N$ 作用得到

\[\cdots+(c_0v_{i,1}+c_1v_{i,2}+\cdots+c_{n_i-1}v_{i,n_i})+\cdots=0.\]

这是一个关于 $N(V)$ 的一组基的一个线性关系,于是 $c_0=\cdots=c_{n_i-1}=0$,从而剩下的线性关系为

\[\cdots+c_{n_i}v_{i,n_i}+\cdots+\sum_{j=q+1}^K d_jw_j=0.\]

而这是一个关于 $\ker N$ 的一组基的一个线性关系,于是 $c_{n_i}=d_{q+1}=\cdots=d_K=0$,从而所有的系数都是 0,这就完成了 Jordan 标准形存在性的证明。



~~~~~~~~~~~~~~~~~~~~~~~~~~~~~~~~~~~~~~~~~~~~~~~~~~~~~~~~~~



分解唯一性的证明:



最后我们还剩下分解唯一性定理的证明,这部分要简单许多,主要是利用了 Jordan 块的一个很特殊的性质:设

\[J_0=\begin{pmatrix}0&1&&\\&\ddots&\ddots&\\&&0&1\\&&&0\end{pmatrix}_{n\times n}\]

是一个 0 特征值的 Jordan 块,则 $J_0^2$ 就是把斜对角线上的 1 向右上方平移一步,$J_0^3$ 就是向右上方平移两步,以此类推,$J_0^{n-1}$ 变成

\[\begin{pmatrix}0&\cdots&1\\&\ddots&\vdots\\&&0\end{pmatrix},\]

最终 $J_0^n=0$。用这个规则我们可以计算出对任何 $\lambda\in\mathbb{C}$ 和 $m\in\mathbb{Z}^+$,$T$ 的 Jordan 标准形中 $m$ 阶 Jordan 块 $J_{\lambda,m}$ 的个数 $n_m$ 来:\[ n_m=\text{rank}(T-\lambda I)^{m-1}-2\cdot\text{rank}(T-\lambda I)^{m}+\text{rank}(T-\lambda I)^{m+1}.\]

道理是这样的:以 $\lambda=0$ 为例子来计算。会算 0 特征值 Jordan 块的个数,你就会算任何特征值的 Jordan 块的个数。设 $T$ 的一个 Jordan 标准形为

\[ T= \left(\bigoplus_{k\geq1}n_k J_{0,k}\right) \bigoplus_{\mu\ne0}J_\mu,\]

那么 $T^m$ 就是

\[T^{m}= \left(\bigoplus_{k\geq1}n_k J_{0,k}^{m}\right) \bigoplus_{\mu\ne0}J_{\mu}^{m}.\]

注意后半部分 $\oplus_{\mu\ne0}J^m_\mu$ 对任何 $m$ 都是保持满秩的,因此这部分的秩始终不变。前面的部分中所有阶数小于等于 $m$ 的 Jordan 块 $J_{0,k}(k\leq m)$ 的 $m$ 次幂都变成了 0 矩阵,$J_{0,m+1}^m$ 的秩是 1; $J_{0,m+2}^m$ 的秩是 2 . . . 依次类推,所以

\[ \text{rank}T^m=n_{m+1}\cdot1+n_{m+2}\cdot2+\cdots +\text{rank}\bigoplus_{\mu\ne0}J_\mu^m.\]

同理

\[ \text{rank}T^{m+1}=n_{m+2}\cdot1+n_{m+3}\cdot2+\cdots +\text{rank}\bigoplus_{\mu\ne0}J_\mu^{m+1}.\]

因此

\[\text{rank}T^m-\text{rank}T^{m+1}=n_{m+1}+n_{m+2}+\cdots,\]

仍然同理

\[\text{rank}T^{m-1}-\text{rank}T^{m}=n_{m}+n_{m+1}+\cdots,\]

所以

\[n_m=\text{rank}T^{m-1}-2\cdot\text{rank}T^{m}+\text{rank}T^{m+1}.\]

现在你可以看到 $n_m$ 的表达式不依赖于具体的基的选择,仅依赖于线性变换自身的相似不变量,所以 $T$ 的 Jordan 标准形在只差一个排列的意义下是唯一的。



~~~~~~~~~~~~~~~~~~~~~~~~~~~~~~~~~~~~~~~~~~~~~~~~~~~~~~~~~~



一个很有意思的问题是,给定

\[J_\lambda=\begin{pmatrix}\lambda&1&&\\&\ddots&\ddots&\\&&\lambda&1\\&&&\lambda\end{pmatrix}_{n\times n}\]

为一个特征值 $\lambda$ 的 Jordan 块,计算其 $k$ 次幂 $J_\lambda^k$ 的 Jordan 标准形。



当 $\lambda\ne0$ 时,

\[J_\lambda^k = \begin{pmatrix}\lambda^k &k\lambda^{k-1}&\ddots &\\&\lambda^k&\ddots&\ddots\\&&\ddots&k\lambda^{k-1}\\&&&\lambda^k\end{pmatrix}.\]

(你知道怎样计算 $J_\lambda^k$ 吗?记住这个技巧:把多项式 $x^k$ 在 $\lambda$ 处 Taloy 展开:\[x^k=(x-\lambda)^k+a_{k-1}(x-\lambda)^{k-1}+a_1(x-\lambda)+a_0,\]然后将 $J_\lambda$ 代入即可。)



和 Jordan 块不可分解性的证明完全一样,我们发现 $J_\lambda^k-\lambda^k I$ 的秩是 $n-1$,因此方程组 $J_\lambda^kX=\lambda^k X$ 的解空间是 1 维的,从而 $J_\lambda^k$ 是不可分解的,因此其 Jordan 标准形只有一块,就是

\[\begin{pmatrix}\lambda^k&1&&\\&\ddots&\ddots&\\&&\lambda^k&1\\&&&\lambda^k\end{pmatrix}_{n\times n}.\]

最有意思的情形发生在 $\lambda=0$ 时。这个时候 Jordan 会均匀的碎裂为一些小的 Jordan 块的和。

这个时候 $J_0$ 是一个移位算子:

\[J_0:\quad v_n\rightarrow v_{n-1}\rightarrow \cdots \rightarrow v_1\rightarrow 0.\]

整个轨道只有一条。但是 $J_0^k$ 则是 $k$ 步 $k$ 步地跳:

\[J_0^k:\quad \left\{ \begin{array}{l} v_n\rightarrow v_{n-k}\rightarrow \cdots \rightarrow0,\\v_{n-1}\rightarrow v_{n-1-k}\rightarrow \cdots\rightarrow 0,\\\cdots\\v_{n-k+1}\rightarrow v_{n-2k+1}\rightarrow \cdots \rightarrow 0.\end{array}\right.\]

所以 $J_0^k$ 有 $k$ 条轨道,每个轨道都是一个 Jordan 块,即 $J_0^k$ 的标准形中有 $k$ 个 Jordan 块。设 $n=qk+r$,这里 $0\leq r< k$,则这 $k$ 个 Jordan 块中有 $r$ 个是 $q+1$ 阶的,$k-r$ 个是 $q$ 阶的。



举个例子就明白了,一个 8 阶的 0 特征值 Jordan 块 $J_0$,$J_0^3$ 的 Jordan 标准形是什么样子的?这个时候 $J_0^3$ 有 3 个轨道 $\{v_8,v_5,v_2\}$, $\{v_7,v_4,v_1 \}$, $\{v_6,v_3\}$,所以 $J_0^3$ 的 Jordan 标准形有 2 个 3 阶的 Jordan 块和 1 个 2 阶的 Jordan 块。

总结一下:零特征值的 Jordan 块的高次幂一定会分裂,而且是尽可能均匀的分裂;非零特征值的 Jordan 块的任意次幂都不会分裂。



一个不可约的代数结构,在某种限制或者扩张的意义下却能均匀的 "碎裂",这是代数学中一个常见而重要的现象。比如设 $f$ 是一个有理数域 $\mathbb{Q}$ 上的不可约多项式,$F$ 是 $Q$ 的一个正规扩域,则如果 $f$ 在 $F$ 上是可约的,那么 $f$ 必然分解成一些次数相同的多项式的乘积:\[f=f_1f_2\cdots f_r,\quad \deg f_1=\cdots=\deg f_r.\]

类似的还有代数数论中素理想的分解,群表示论中不可约表示(在诱导和限制下)的分解,代数几何中不可约代数簇的分解等等。
 

May 2

问题设$A,B\in M_{n}(C)$,求证:
\[ ||AB-BA||_{F}\leq \sqrt{2}||A||_{F}||B||_{F}  \]
证明:设
\[ A=\text{diag}(a_{1},a_{2},\cdots,a_{n})\qquad B=(b_{ij}) \]
\[ AB-BA=\text{diag}(a_{1},a_{2},\cdots,a_{n})B-B\text{diag}(a_{1},a_{2},\cdots,a_{n})=((a_{i}-a_{j})b_{ij})\]

\begin{align*}
 ||AB-BA||_{F}^{2}&=\sum_{i,j}|a_{i}-a_{j}||b_{ij}|^{2}\\
 &\leq \sum_{i,j}2\left(|a_{1}|^2+|a_{2}|^2+\cdots+|a_{n}|^{2}\right)|b_{ij}|^{2}\\
 &=2 ||A||_{F}^{2}||B||_{F}^{2}
\end{align*}
一般地,用A的奇异值SVD分解为
\[ A=U\text{diag}(a_{1},a_{2},\cdots,a_{n})V \]
其中$U,V$为酉矩阵,则变成上面的情形。

Mar 2

1.设$A = \left( {\begin{array}{*{20}{c}}
1&2&0&0\\
1&3&0&0\\
0&0&0&2\\
0&0&{ - 1}&0
\end{array}} \right)$且$\left[\left(\frac{1}{2}A\right)^{*}\right]^{-1}BA=6AB+12E$,求$B$.


解:我们有$A^{*}A=AA^{*}=|A|E$,而
\[A = \left( {\begin{array}{*{20}{c}}
X&{}\\
{}&Y
\end{array}} \right)\]
其中
\[ X = \left( {\begin{array}{*{20}{c}}
1&2\\
1&3
\end{array}} \right)\qquad Y = \left( {\begin{array}{*{20}{c}}
0&2\\
{-1}&0
\end{array}} \right)\]

\[ |A|=|X||Y|=1\cdot 2=2\]

\[ BA=3\cdot |A|B+6A^{*}=6B+6A^{*}\]
\[\Rightarrow B(A-6E)=6A^{*}\]
\[\Rightarrow B(A-6E)A=12E \]
\[ B=12[(A-6E)A]^{-1} \]

\[\left( {A - 6E} \right)A = \left( {\begin{array}{*{20}{c}}
{ - 5}&2&{}&{}\\
1&{ - 3}&{}&{}\\
{}&{}&{ - 6}&2\\
{}&{}&{ - 1}&{ - 6}
\end{array}} \right)\left( {\begin{array}{*{20}{c}}
1&2&{}&{}\\
1&3&{}&{}\\
{}&{}&0&2\\
{}&{}&{ - 1}&0
\end{array}} \right) = \left( {\begin{array}{*{20}{c}}
{ - 3}&{ - 4}&{}&{}\\
{ - 2}&{ - 7}&{}&{}\\
{}&{}&{ - 2}&{ - 12}\\
{}&{}&6&{ - 2}
\end{array}} \right)\]
\[{\left( {\begin{array}{*{20}{c}}
{ - 3}&{ - 4}&{}&{}\\
{ - 2}&{ - 7}&{}&{}\\
{}&{}&{ - 2}&{ - 12}\\
{}&{}&6&{ - 2}
\end{array}} \right)^{ - 1}} = \left( {\begin{array}{*{20}{c}}
{ - \frac{7}{{13}}}&{\frac{4}{{13}}}&{}&{}\\
{\frac{2}{{13}}}&{ - \frac{3}{{13}}}&{}&{}\\
{}&{}&{ - \frac{1}{{38}}}&{\frac{6}{{38}}}\\
{}&{}&{ - \frac{3}{{38}}}&{ - \frac{1}{{38}}}
\end{array}} \right)\]
所以
\[B = 12\left( {\begin{array}{*{20}{c}}
{ - \frac{7}{{13}}}&{\frac{4}{{13}}}&{}&{}\\
{\frac{2}{{13}}}&{ - \frac{3}{{13}}}&{}&{}\\
{}&{}&{ - \frac{1}{{38}}}&{\frac{6}{{38}}}\\
{}&{}&{ - \frac{3}{{38}}}&{ - \frac{1}{{38}}}
\end{array}} \right)\]
二、计算$D = \left| {\begin{array}{*{20}{c}}
{{s_0}}&{{s_1}}& \cdots &{{s_{n - 1}}}\\
{{s_1}}&{{s_2}}& \cdots &{{s_n}}\\
 \vdots & \vdots & \cdots & \vdots \\
{{s_n}}&{{s_{n + 1}}}& \cdots &{{s_{2n - 1}}}
\end{array}\begin{array}{*{20}{c}}
1\\
x\\
 \vdots \\
{{x^n}}
\end{array}} \right|$,其中${s_k} = x_1^k + x_2^k +  \cdots x_n^k$


解:注意到
\begin{align*}
D &= \left| {\begin{array}{*{20}{c}}
{1 + 1 +  \cdots  + 1}&{{x_1} + {x_2} +  \cdots  + {x_n}}& \cdots &{x_1^{n - 1} + x_2^{n - 1} +  \cdots  + x_n^{n - 1}}&1\\
{{x_1} + {x_2} +  \cdots  + {x_n}}&{x_1^2 + x_2^2 +  \cdots  + x_n^2}& \cdots &{x_1^n + x_2^n +  \cdots  + x_n^n}&x\\
 \vdots & \vdots & \cdots & \vdots & \vdots \\
 \vdots & \vdots & \cdots & \vdots & \vdots \\
{x_1^n + x_2^n +  \cdots  + x_n^n}&{x_1^{n + 1} + x_2^{n + 1} +  \cdots  + x_n^{n + 1}}& \cdots &{x_1^{2n - 1} + x_2^{2n - 1} +  \cdots  + x_n^{2n - 1}}&{{x^n}}
\end{array}} \right|\\
 &= \left| {\begin{array}{*{20}{c}}
1&1& \cdots &1&1\\
{{x_1}}&{{x_2}}& \cdots &{{x_n}}&x\\
{x_1^2}&{x_2^2}& \cdots &{x_n^2}&{{x^2}}\\
 \vdots & \vdots & \cdots & \vdots & \vdots \\
{x_1^n}&{x_2^n}& \cdots &{x_n^n}&{{x^n}}
\end{array}} \right| \cdot \left| {\begin{array}{*{20}{c}}
1&{{x_1}}&{x_1^2}& \cdots &{x_1^{n - 1}}&0\\
1&{{x_2}}&{x_2^2}& \cdots &{x_2^{n - 1}}&0\\
1&{{x_3}}&{x_3^2}& \cdots &{x_3^{n - 1}}&0\\
 \vdots & \vdots & \vdots & \cdots & \vdots &0\\
1&{{x_n}}&{x_n^2}& \cdots &{x_n^{n - 1}}&0\\
0&0&0& \cdots &0&1
\end{array}} \right|\\
&=V(x_{1},x_{2},\cdots,x_{n},x)\cdot V(x_{1},x_{2},\cdots,x_{n})
\end{align*}
三、有$\alpha_{1},\alpha_{2},\cdots,\alpha_{s},\alpha_{s+1}$,且$\beta_{i}=\alpha_{i}+t_{i}\alpha_{s+1},i=1,2,\cdots,s$, 证明如果$\beta_{1},\beta_{2},\cdots,\beta_{s}$,线性无关,则$\alpha_{1},\alpha_{2},\cdots,\alpha_{s+1}$必定线性无关。


五、设$sl_{n}(F)$是$M(F)$上$A,B$矩阵满足$AB-BA$生成的子空间,证明$\dim(sl_{n}(F))=n^2-1$.


证明:
\[ sl_{n}(F)=\{AB-BA|A,B\in M(F)\} \]
对任意的$X\in sl_{n}(F)$,存在$A,B\in M(F)$,使得
\[ X=AB-BA \]
我们注意到
\[ \text{tr}(X)=\text{tr}(AB-BA)=0 \]
假设
\[X = \left( {\begin{array}{*{20}{c}}
{{a_{11}}}&{{a_{12}}}& \cdots & \cdots &{{a_{1n}}}\\
{{a_{21}}}&{{a_{22}}}& \cdots & \cdots &{{a_{2n}}}\\
 \vdots & \vdots & \vdots & \vdots & \vdots \\
 \vdots & \vdots & \vdots & \vdots & \vdots \\
{{a_{n1}}}&{{a_{n2}}}& \cdots & \cdots &{{a_{nn}}}
\end{array}} \right)\]
则有$a_{11}+a_{22}+\cdots+a_{nn}=0$,于是,当$a_{22},a_{33},\cdots,a_{nn}$确定了,$a_{11}$自然就确定了。下证明
\[{E_{ij}} = \left( {\begin{array}{*{20}{c}}
{}&{}&{}&{}&{}\\
{}&{}&{}&{}&{}\\
{}&{}&{}&1&{}\\
{}&{}&{}&{}&{}\\
{}&{}&{}&{}&{}
\end{array}} \right)(i\neq j)\]
\[{E_{ii}} = \left( {\begin{array}{*{20}{c}}
{ - 1}&{}&{}&{}&{}&{}\\
{}& \ddots &{}&{}&{}&{}\\
{}&{}&1&{}&{}&{}\\
{}&{}&{}& \ddots &{}&{}\\
{}&{}&{}&{}& \ddots &{}\\
{}&{}&{}&{}&{}&0
\end{array}} \right)\qquad (i\geq 2)\]
这里$E_{ij}$表示第$i$行第$j$列为$1$,其他为$0$的矩阵,$E_{ii}$表示第一行第一列为$-1$,第$i$行第$i$列为$1$,其余元素为$0$,的矩阵。不难计算发现所有的这2类矩阵数目为$n^2-1$,下证明它们可以构成$sl_{n}(F)$的一组基。
\[ \sum_{ij\neq 1}k_{ij}E_{ij}=0\]
\[\left( {\begin{array}{*{20}{c}}
{ - \sum\limits_{i = 2}^n {{k_{ii}}} }&{{k_{12}}}&{{k_{13}}}& \cdots & \cdots &{{k_{1n}}}\\
{{k_{21}}}&{{k_{22}}}&{{k_{23}}}& \cdots & \cdots &{{k_{2n}}}\\
 \cdots & \cdots & \cdots & \cdots & \cdots & \cdots \\
 \vdots & \vdots & \vdots & \vdots & \vdots & \vdots \\
 \vdots & \vdots & \vdots & \vdots & \vdots & \vdots \\
{{k_{n1}}}&{{k_{n2}}}& \cdots & \cdots & \cdots &{{k_{nn}}}
\end{array}} \right) = 0\]
显然有$k_{ij}=0$,因此,这$n^2-1$个矩阵线性无关,而对任意的$X\in sl_{n}(F)$,有
\[ X=\sum_{ij\neq 1}a_{ij}E_{ij}\]
故$E_{12},\cdots,E_{1n},\cdots E_{nn}$为$sl_{n}(F)$的一组基。马上得到$\dim(sl_{n}(F))=n^2-1$。

 


六、设数域$K$上的$n$维线性空间$V$到$m$维线性空间上的所有线性映射组成线性空间$Hom_{k}(V,V')$.证明
(1)$Hom_{k}(V,V')$是线性空间;
(2)$Hom_{k}(V,V')$的维数为$mn$.


证明:(1)先验证线性空间的8条运算法则。对任意的$\alpha\in V$,及$\forall f,g,h\in Hom_{k}(V,V')$
1
$(f+g)(\alpha)=f(\alpha)+g(\alpha)=g(\alpha)+f(\alpha)=(g+f)(\alpha)\qquad \forall f,g\in Hom_{k}(V,V')$
2
$((f+g)+h)(\alpha)=(f+g)(\alpha)+h(\alpha)=f(\alpha)+g(\alpha)+h(\alpha)=f(\alpha)+(g+h)(\alpha)=(f+(g+h))(\alpha) $
3在$Hom_{k}(V,V')$中存在一个变换$0$,使得$f+0=f,\forall f\in Hom_{k}(V,V')$.
4.对于$Hom_{k}(V,V')$中的每个元素$f$,都存在$-f\in Hom_{k}(V,V')$,使得$f+(-f)=0$.
5.对于$Hom_{k}(V,V')$中的每个元素$f$,有$1f=f$
6.$k(lf)(\alpha)=klf(\alpha)=(kl)f(\alpha),\forall k,l\in K,\forall \alpha\in V$.
7.$(k+l)f(\alpha)=kf(\alpha)+lf(\alpha),\forall k,l \in K,\forall \alpha \in V$.
8.$(k(f+g))(\alpha)=kf(\alpha)+kg(\alpha),\forall k\in K,\forall \alpha \in V$.
所以$Hom_{k}(V,V')$是线性空间;
(2)我们在$V$中找一组基,记为$\eta_{1},\eta_{2},\cdots,\eta_{n}$,$V'$中找一组基,记为$\nu_{1},\nu_{2},\cdots,\nu_{m}$,则对任意的$\alpha\in V$,
\[ \alpha=\sum_{k=1}x_{k}\eta_{k} \]
\[ f(\alpha)=\sum_{k=1}^{n}x_{k}f(\eta_{k})\]
而对于$f(\eta_{k})\in V'$,
\[ f(\eta_{k})=\sum_{j=1}^{m}y_{kj}\nu_{j}\]
所以
\[ f(\alpha)=\sum_{k=1}^{n}\sum_{j=1}^{m}x_{k}y_{kj}\nu_{j}\]
\[ f(\eta_{1},\eta_{2},\cdots,\eta_{n})=(\nu_{1},\nu_{2},\cdots,\nu_{m})\left( {\begin{array}{*{20}{c}}
{{y_{11}}}&{{y_{12}}}& \cdots & \cdots & \cdots &{{y_{1n}}}\\
{{y_{21}}}&{{y_{22}}}& \cdots & \cdots & \cdots &{{y_{2n}}}\\
 \vdots & \vdots & \vdots & \vdots & \vdots & \vdots \\
 \vdots & \vdots & \vdots & \vdots & \vdots & \vdots \\
 \vdots & \vdots & \vdots & \vdots & \vdots & \vdots \\
{{y_{m1}}}&{{y_{m2}}}& \cdots & \cdots & \cdots &{{y_{mn}}}
\end{array}} \right)\]

\[ A=\left( {\begin{array}{*{20}{c}}
{{y_{11}}}&{{y_{12}}}& \cdots & \cdots & \cdots &{{y_{1n}}}\\
{{y_{21}}}&{{y_{22}}}& \cdots & \cdots & \cdots &{{y_{2n}}}\\
 \vdots & \vdots & \vdots & \vdots & \vdots & \vdots \\
 \vdots & \vdots & \vdots & \vdots & \vdots & \vdots \\
 \vdots & \vdots & \vdots & \vdots & \vdots & \vdots \\
{{y_{m1}}}&{{y_{m2}}}& \cdots & \cdots & \cdots &{{y_{mn}}}
\end{array}} \right)\]
构造映射$\phi:f\to A$,则$\varphi$是从$Hom_{n}(V,V')$到$M_{m\times n}$上的映射。则有
\[ \phi(f+g)=\phi(f)+\phi(g) \]
\[ \phi(kf)=k\phi(f) \]
对于任意的$f,g$满足
\[ \phi(f)=\phi(g)\]
则有
\[ f=g \]
对任意的$A\in M_{m\times n}$,存在$f\in Hom_{n}(V,V')$,使得$\phi(f)=A$,所以
\[ Hom_{n}(V,V')\cong M_{m\times n}\]
而我们知道$\dim(M_{m\times n})=mn$,故$Hom_{k}(V,V')$的维数为$mn$.

七、已知
\[F = \left( {\begin{array}{*{20}{c}}
0&{}&{}&{}&{}&{ - {c_0}}\\
1&0&{}&{}&{}&{ - {c_1}}\\
{}&1&0&{}&{}& \vdots \\
{}&{}&1& \ddots &{}&{ - {c_{n - 3}}}\\
{}&{}&{}& \ddots &0&{ - {c_{n - 2}}}\\
{}&{}&{}&{}&1&{ - {c_{n - 1}}}
\end{array}} \right)\]
(1)求$F$的特征多项式$f(x)$与最小多项式$m(x)$;
(2)求所有与$F$可交换的矩阵.


八、设$\varphi$是复数域上的线性变换,$\epsilon$为恒等变换,$\lambda_{0}$为$\varphi$的一个特征值,$\lambda_{0}$在$\varphi$的最小多项式中的重数$$m_{0}=\min_{k}\{k\in\mathbf{N}^{+}|\text{ker}(\lambda_{0}\epsilon-\varphi)^{k}=\text{ker}(\lambda_{0}\epsilon-\varphi)^{k+1}\}$$

 

Dec 7
设$n$阶矩阵$A,B$可交换,证明:$$r(A+B)\leq r(A)+r(B)-r(AB)$$
证明:先证一个引理:
设$A,B$是数域$K$上两个$n\times n$矩阵且$AB=BA$.又设

\[ C=\left[
\begin{array}{cc}
A \\
B \\
\end{array}\right]\]

则有

\[ r(A)+r(B)\geq r(C)+r(AB) \]
证明:考察奇次线性方程组$AX=0$和$BX=0$,它们的全部公共解向量恰好为$CX=0$的解向量。故设$CX=0$的一个基础解系是
\[ \varepsilon_{1},\varepsilon_{2},\cdots\varepsilon_{r}\]
这里$r=n-r(C)$,我们知道,它可以分别扩充到$AX=0$的一个基础解系
\[ \varepsilon_{1},\varepsilon_{2},\cdots\varepsilon_{r},\eta_{1},\eta_{2}\cdots,\eta_{s}\]
和$BX=0$的一个基础解系
\[ \varepsilon_{1},\varepsilon_{2},\cdots\varepsilon_{r},\omega_{1},\omega_{2}\cdots,\omega_{t}\]
这里$r+s=n-r(A),r+t=n-r(B)$.而线性组合
\[ a_{1}\eta_{1}+\cdots+a_{s}\eta_{s}\]
是$AX=0$的解向量,但当它非零时它不是$BX=0$的解向量,否则它就是两者的公共解向量,即有$CX=0$,从而可以被$CX=0$的解系线性表示
\[ a_{1}\eta_{1}+\cdots+a_{s}\eta_{s}=k_{1}\varepsilon_{1}+\cdots k_{r}\varepsilon_{r}\]
此时由于$\varepsilon_{1},\varepsilon_{2},\cdots\varepsilon_{r},\eta_{1},\eta_{2}\cdots,\eta_{s}$线性无关,所以得到$a_{1}=a_{2}=\cdots a_{s}=k_{1}=\cdots k_{r}=0$,这与$a_{1}\eta_{1}+\cdots+a_{s}\eta_{s}$非零矛盾,由此可见
\[ \varepsilon_{1},\varepsilon_{2},\cdots\varepsilon_{r},\eta_{1},\eta_{2}\cdots,\eta_{s},\omega_{1},\omega_{2}\cdots,\omega_{t}\]
线性无关。
现在,上面都是$ABX=BAX=0$的解向量,它们又线性无关,故$r+s+t\leq n-r(AB)$即
\[ (r+s)+(r+t)-r=n-r(A)+n-r(B)-(n-r(C))\leq n-r(AB)\]
\[ \Leftrightarrow r(C)+r(AB)\leq r(A)+r(B) \]
引理证明完毕!
 
最后回到原题,只要注意到

\[A + B = \left[{{E_n},{E_n}}\right]\left[ {\begin{array}{*{20}{c}}
A\\
B
\end{array}} \right]\]

\[ r(A+B)\leq r(C)\]
故马上有
\[ r(A+B)\leq r(A)+r(B)-r(AB) \]
Done!
Apr 20

问题:设$A$为$m\times n$阶矩阵,$B$为$n\times m$阶,$m>n$,求证
\[ |\lambda I_{m}-AB|=\lambda^{m-n}|\lambda I_{n}-BA| \]
证明
当$\lambda=0$的时候,结论显然成立。下面证明$\lambda\neq 0$的情况。
注意到矩阵
\[ \left(
\begin{array}{cc}
\lambda E_{m} & A \\
B & E_{n} \\
\end{array}
\right)\]
有2种打洞方式。就是
\[ \left(
\begin{array}{cc}
E_{m} & -A \\
O & E_{n} \\
\end{array}
\right)\left(
\begin{array}{cc}
\lambda E_{m} & A \\
B & E_{n} \\
\end{array}
\right)
\left(\begin{array}{cc}
E_{m} & O \\
-B & E_{n} \\
\end{array}
\right)=
\left(
\begin{array}{cc}
\lambda E_{m}-AB & O \\
O & E_{n} \\
\end{array}
\right)\]

\[ \left(
\begin{array}{cc}
E_{m} & O \\
-\frac{1}{\lambda} & E_{n} \\
\end{array}
\right)
\left(
\begin{array}{cc}
\lambda E_{m} & A \\
B & E_{n} \\
\end{array}
\right)
\left(
\begin{array}{cc}
E_{m} & -\frac{1}{\lambda}A \\
O & E_{n} \\
\end{array}
\right)=
\left(
\begin{array}{cc}
\lambda E_{m} & O \\
O & E_{n}-\frac{1}{\lambda}BA\\
\end{array}
\right)\]
对上面两个式子取行列式,马上就可以得到
\[ |\lambda I_{m}-AB|=\lambda^{m-n}|\lambda I_{n}-BA| \]
$\square$  

我代数弱,大家不要笑啊。